Đến nội dung

tquangmh nội dung

Có 235 mục bởi tquangmh (Tìm giới hạn từ 30-03-2020)



Sắp theo                Sắp xếp  

#693379 Muốn luyện thi HSG từ năm lớp 10 thì nên bắt đầu từ đâu ?

Đã gửi bởi tquangmh on 19-09-2017 - 20:56 trong Kinh nghiệm học toán

nếu trường chuyên em hướng vào đó cũng được nhưng trường THPT không chuyên thì thầy cô nói đúng đấy hồi lớp 10 chị cũng từng thích mấy chuyện hsg này nhưng giờ nhận ra rồi nên có giới hạn thôi

 

Em cảm ơn dòng tư vấn của chị ạ. :v 




#693345 Muốn luyện thi HSG từ năm lớp 10 thì nên bắt đầu từ đâu ?

Đã gửi bởi tquangmh on 18-09-2017 - 23:38 trong Kinh nghiệm học toán

     Như tiêu đề trên, em mong anh chị có thể "truyền" kinh nghiệm luyện thi HSG THPT cho em.  :biggrin: 

 

     Khi còn học THCS thì tỉnh em không dạy nhiều về Số học và Tổ hợp nên ngay khi bước lên cấp 3 em thấy hoảng sợ quá ạ :wacko:  :wacko: .  

 

Có vài câu hỏi mà em thắc mắc mong được giải đáp :

 

     $1/$ Những kiến thức em có thể dùng được trong kì thi Olympic 30/4 giới hạn đến đâu ạ ?

 

     $2/$ Về phần thi HSG THPT thì có những kiến thức nào em áp dụng mà không cần chứng minh lại ? (về các mảng như Hình học phẳng, Số học, Phương trình hàm, ...)

 

     $3/$ Các anh lớp trên khuyên em lên cấp 3 ít học thêm lại nên đa phần em vẫn tự học. Tuy nhiên em chưa đụng đến Số học và Tổ hợp. Vậy em nên bắt đầu 2 phần toán này từ đâu ạ ? (ý em là ở từng mảng như Số học, Tổ hợp thì em nên bắt đầu học những kiến thức nào trước làm nền ạ ??)

 

(Trường THPT em đang học thầy cô nghiêng về thi THPTQG nhiều hơn nên em ít có điều kiện hỏi các thầy cô ạ !!  :( )

 

 Em xin cảm ơn. :icon6:  




#651305 Tìm các số nguyên dương $a, b, c> 1.$

Đã gửi bởi tquangmh on 25-08-2016 - 23:33 trong Số học

Tìm các số nguyên dương $a, b, c> 1$ đôi một khác nhau thỏa mãn: $abc-1\vdots (a-1)(b-1)(c-1).$

 

 

Do vai trò của $a;b;c$ là như nhau nên giả sử : $a\leq b\leq c$. 

Đặt :$\left\{\begin{matrix} x=a−1\\ y=b−1 \\ z=c−1 \end{matrix}\right.$ với  $0< x\leq y\leq z$ và $x;y;z \in \mathbb{Z}^{+}$.

Ta có : 

$(x+1)(y+1)(z+1)−1\vdots xyz$

$\Leftrightarrow xyz+xy+yz+zx+x+y+z\vdots xyz$

$\Leftrightarrow \frac{xyz+xy+yz+zx+x+y+z}{xyz} \in \mathbb{Z}^{+}$

$\Leftrightarrow f(x,y,z)=\frac{1}{x}+\frac{1}{y}+\frac{1}{z}+\frac{1}{xy}+\frac{1}{yz}+\frac{1}{zx} \in \mathbb{Z}^{+}$

 

Để ý rằng : Nếu $a>b;a,b \in \mathbb{Z}^{+}\Rightarrow \frac{1}{a}< \frac{1}{b}$.

Do đó : 

$f(x,y,z)\leq f(1,2,3)\approx 2,8(3)\Rightarrow f(x,y,z) \in \left \{ 1;2 \right \}$

mà : $f(3,4,5)\approx 0,98(3) < 1 \Rightarrow x \in \left \{ 1;2 \right \}$

 

* TH1 : $\left\{\begin{matrix} f(x,y,z)=1\\x=1 \end{matrix}\right.$. Có : 

$f(1,y,z)=1+\frac{2}{y}+\frac{2}{z}+\frac{1}{yz}=1$

$\Leftrightarrow \frac{2}{z}+\frac{2}{y}+\frac{1}{yz}=0$ (vô lí vì $y;z\in \mathbb{Z}^{+}$ )

Lí luận tương tự : 

TH2 : $\left\{\begin{matrix} f(x,y,z)=1\\ x=2 \end{matrix}\right.$. $\rightarrow$ $(x=2;y=4;z=14)$ $\rightarrow$ $(a=3;b=5;c=15)$.

* TH3 : $\left\{\begin{matrix} f(x,y,z)=2\\ x=1 \end{matrix}\right.$. $\rightarrow$ $(x=1;y=3;z=7)\rightarrow (a=2;b=4;c=8)$.

* TH4 : $\left\{\begin{matrix} f(x,y,z)=1\\ x=2 \end{matrix}\right.$. $\rightarrow$ vô nghiệm.

 

 

Vậy : $(a;b;c)=(3;5;15);(2;4;8)$ cùng các hoán vị. 

$\blacksquare$




#635242 Chứng minh rằng $abc(a+b+c)+2\geq 2(ab+bc+ca)$

Đã gửi bởi tquangmh on 24-05-2016 - 19:30 trong Bất đẳng thức và cực trị

Đã giải ở link này. http://diendantoanho...e-5#entry630624

 

Gần cuối trang 5, bài 51




#632481 n chia hết cho 9 và n+1 chia hết cho 25

Đã gửi bởi tquangmh on 11-05-2016 - 16:16 trong Số học

Minh hieu r, tks b nhieu nha.
Các bài khác b giải dc ko?

 

Câu 1b : Làm tương tự như câu a), nhưng lúc này ta lấy kết quả n + 2 = 9(25f + 11)  + 2 chỉ cần xét chia hết cho 4.

Câu 3 : Mình chưa học về phi.  :P Bạn có thể đăng lại bài này đề các anh chị giải cho. Giống câu 2 đó.




#632443 n chia hết cho 9 và n+1 chia hết cho 25

Đã gửi bởi tquangmh on 11-05-2016 - 12:06 trong Số học

tks b nhé :)
B cho mình hỏi, tại sao lại chọn r=4 mà ko chọn số khác vay ?

 

khi đã xác định đc 25t - 1 chia hết cho 9 thì mình xét các số t = 9f + r (r =0;1;2;...;8)

Thử với r = 1 vào : 

 25t - 1 = 25(9f + 1) - 1 = 25.9f + 25 - 1 = 25.9f + 24 mà 24 ko chia hết cho 9 nên r = 1 ko thỏa mãn 25t - 1 chia hết cho 9.

Thử lần lượt thì thấy r = 4 thỏa mãn. (99 chia hết cho 9) 




#632405 n chia hết cho 9 và n+1 chia hết cho 25

Đã gửi bởi tquangmh on 11-05-2016 - 00:09 trong Số học

Bài 1 : 

Theo đề bài, ta có : 

$n = 9k$   (k là số tự nhiên)

$n + 1 = 25t$   (t là số tự nhiên)

Từ đó, ta suy ra : 

$n = 25t - 1 = 9k$

$\Rightarrow 25t - 1\vdots 9$

Xét $t = 9f + r$ (f, r là số tự nhiên; $r = 0;1;2;3;...;8$)

Thử với các giá trị của $r$, ta chọn đc $r = 4$.

Do đó : $n + 1 = 25(9f + 4) = 225f + 100$ 

Vậy : $n = 9(25f + 11)$ với $f$ là số tự nhiên. 




#631848 CMR:$(a+b+c)(\frac{1}{a}+\frac{1...

Đã gửi bởi tquangmh on 07-05-2016 - 23:14 trong Bất đẳng thức và cực trị

Biểu thức này phải âm chứ bạn?

 

Cảm ơn bạn đã góp ý. Mình ko đọc rõ đề. 




#631823 CMR:$(a+b+c)(\frac{1}{a}+\frac{1...

Đã gửi bởi tquangmh on 07-05-2016 - 21:48 trong Bất đẳng thức và cực trị

Cho a,b,c là độ dài ba cạnh tam giác. CMR:$(a+b+c)(\frac{1}{a}+\frac{1}{b}+\frac{1}{c})+\frac{3(a-b)(b-c)(c-a)}{abc}\geq 9$

 

_ Ko mất tính tống quát, do a, b, c là độ dài 3 cạnh tam giác nên có thể giả sử : $a\geq b\geq c$ $\Rightarrow$ $\frac{3(a-b)(b-c)(c-a)}{abc}\geq 0$

_Còn : $(a+b+c)(\frac{1}{a}+\frac{1}{b}+\frac{1}{c})\geq (3.\sqrt{abc}).(\frac{3}{\sqrt{abc}})=9$

_ Dấu bằng khi tam giác đó đều.

 

P/s : Mình thấy có gì đó sai sai, mong mọi người chỉ bảo, sửa chữa giúp mình.




#631475 C/m các bt sau không thể có cùng giá trị âm: ab-a-b+1 ; bc-b-c+1 và ac-a-c+1

Đã gửi bởi tquangmh on 05-05-2016 - 21:23 trong Các dạng toán khác

1.C/m các bt sau không thể có cùng giá trị âm:

ab-a-b+1 ; bc-b-c+1 và ac-a-c+1

2.C/m đa thức x2014+x2012+1 chia hết cho đa thức x2+x+1

 

 

Bài 1 : 

$ab-a-b+1=(a-1)(b-1);bc-b-c+1=(b-1)(c-1);ca-c-a+1=(a-1)(c-1)$

$\Rightarrow (ab-a-b+1)(bc-b-c+1)(ca-c-a+1)=(a-1)^{2}(b-1)^{2}(c-1)^{2}\geq 0$        $(1)$

Nếu các biểu thức trên cùng âm thì $(1)$ ko xảy ra (vô lí). Vậy, các biểu thức trên ko thể cùng âm

 

Bài 2 : đề có bị sai ko bạn ?? 

 

 




#631064 min $P=3a+2b+c$

Đã gửi bởi tquangmh on 03-05-2016 - 21:20 trong Bất đẳng thức và cực trị

$2abc=3a^{2}+4b^{2}+5c^{2}$ nên $abc>0$

 

$abc> 0$ nhỡ mà  $a< 0;b> 0;c< 0$ thì $a^{6}b^{8}c< 0$ rồi anh !




#631053 min $P=3a+2b+c$

Đã gửi bởi tquangmh on 03-05-2016 - 21:04 trong Bất đẳng thức và cực trị

Theo AM-GM: $$2abc=3a^{2}+4b^{2}+5c^{2}\geq 12\sqrt[12]{a^{6}b^{8}c}\Leftrightarrow a^{3}b^{2}c\geq 46656$$

Do đó: $$3a+2b+c\geq 6\sqrt[6]{a^{3}b^{2}c}=36$$

 

Đầu bài là các số thực bất kì mà anh.  :mellow:  




#630902 Tìm Max của $A=2(x^{3}+y^{3})+3(x^{2}+y^...

Đã gửi bởi tquangmh on 02-05-2016 - 22:01 trong Bất đẳng thức và cực trị

Cho các số thực x,y thỏa mãn $x+y+4=0$. Tìm Max của

$A=2(x^{3}+y^{3})+3(x^{2}+y^{2})+10xy$

 

 

Có : 

$A=2(x^{3}+y^{3})+3(x^{2}+y^{2})+10xy$

$\Leftrightarrow A=2(x+y)(x^{2}-xy+y^{2})+3(x^{2}+y^{2})+10xy$

$\Leftrightarrow A=-5(x^{2}+y^{2})+18xy$

Phần : $-5(x^{2}+y^{2})$ đánh giá max bằng BĐT Bunhia

Phần : $18xy$ dựa vào BĐT đc chứng minh bằng tương đương : $xy\leq \frac{(x+y)^{2}}{4}$




#630271 $2(a^{4}+b^{4})\geq ab^{3}+a^{3...

Đã gửi bởi tquangmh on 29-04-2016 - 23:12 trong Bất đẳng thức và cực trị

CM

1. $2(a^{4}+b^{4})\geq ab^{3}+a^{3}b+2a^{2}b^{2}$   $\forall a,b$

 

 

Chém câu dễ nhất :D : 

Ta có : 

+) $a^{4}+b^{4} \geq ab^{3}+a^{3}b \Leftrightarrow (a-b)^{2}(a^{2}+ab+b^{2}) \geq 0$ 

+) $a^{4}+b^{4} \geq 2a^{2}b^{2} \Leftrightarrow (a^{2}-b^{2})^{2} \geq 0$




#630266 Đồng dư thức

Đã gửi bởi tquangmh on 29-04-2016 - 22:54 trong Số học

Tìm n biết 2n chia 7 dư 1

 

Bài toán đưa về : Tìm $n\in \mathbb{N}$ để $2^{n}\equiv 1(mod7)$.

Giải : 

_ Xét : $n=3k(k\in \mathbb{N})$. 

Ta có : $2^{3k}\equiv 1(mod7)\Leftrightarrow 8^{k}\equiv 1(mod7)$ (luôn đúng). Vậy ta đc 1 nghiệm là $n=3k$.

 

_ Xét : $n=3k+1(k\in \mathbb{N})$.

Ta có : $2^{3k+1}\equiv 1(mod7)\Leftrightarrow (2.8^{k}-1)\vdots 7\Leftrightarrow [8^{k}+(8^{k}-1)]\vdots 7$  $(1)$

Dễ thấy : $(8^{k}-1)\vdots 7$ và $8^{k}$ không chia hết cho 7 nên $(1)$ vô lí.

 

_ Xét : $n=3k+2(k\in \mathbb{N})$.

Ta có : $2^{3k+2}\equiv 1(mod7)\Leftrightarrow (4.8^{k}-1)\vdots 7\Leftrightarrow [3.8^{k}+(8^{k}-1)]\vdots 7$.    $(2)$

Lập luận tương tự trường hợp $n=3k+1$ và để ý thêm rằng : $(3;7)=1$. Ta thấy $(2)$ cũng vô lí.

 

Vậy :  $n=3k(k\in \mathbb{N})$ 

  




#630106 Tìm GTNN của $A= 2016xy-yz-zx$

Đã gửi bởi tquangmh on 28-04-2016 - 21:32 trong Bất đẳng thức và cực trị

bạn tham khảo : http://diendantoanho...ca/#entry630020




#629939 Tìm $Min$ và $Max$ của $Q=x+y+z$

Đã gửi bởi tquangmh on 27-04-2016 - 22:41 trong Bất đẳng thức và cực trị

cho x,y,z là các số thực thoả mãn: $y^{2}+yz+z^{2}=1-\frac{3}{2}x^{2}$ .Tim Min,Max của: 

            Q=x+y+z

 

Ta có : 

$y^{2}+yz+z^{2}=1-\frac{3x^{2}}{2}$

$\Leftrightarrow 2y^{2}+2yz+2z^{2}+3x^{2}=2$

$\Leftrightarrow 2=x^{2}+y^{2}+z^{2}+(x^{2}+y^{2})+(z^{2}+x^{2})+2yz\geq x^{2}+y^{2}+z^{2}+2xy+2zx+2yz$

$\Leftrightarrow (x+y+z)^{2}\leq 2$

$\Leftrightarrow -\sqrt{2}\leq x+y+z\leq \sqrt{2}$

Dấu "=" : ....

* Giải thích : $x^{2}+y^{2}\geq 2xy\Leftrightarrow (x-y)^{2}\geq 0$ còn $z^{2}+x^{2}\geq 2zx$ tương tự. Hai BDT này đúng với mọi số thực x, y, z.

 

P/s : Bài này bạn tpdtthltvp có giải ở đâu rồi nhưng mình quên link.




#629910 $P=\frac{x^4}{y+z}+\frac{y^4}...

Đã gửi bởi tquangmh on 27-04-2016 - 21:39 trong Bất đẳng thức và cực trị

Cho x,y,z >0 và $x^3+y^3+z^3=2$. Tìm GTNN

$P=\frac{x^4}{y+z}+\frac{y^4}{x+z}+\frac{z^4}{x+y}$

 

* Bổ đề : $a^{3}+b^{3}+c^{3}\geq a^{2}b+b^{2}c+c^{2}a$ và $a^{3}+b^{3}+c^{3}\geq ab^{2}+bc^{2}+ca^{2}$

Dễ chứng minh bằng Cauchy

 

Ta có : $P=\frac{x^{4}}{y+z}+\frac{y^{4}}{z+x}+\frac{z^{4}}{x+y}=\frac{x^{6}}{x^{2}y+x^{2}z}+\frac{y^{6}}{y^{2}x+y^{2}z}+\frac{z^{6}}{z^{2}x+z^{2}y}\geq \frac{(x^{3}+y^{3}+z^{3})^{2}}{(x^{2}y+y^{2}z+z^{2}x)+(xy^{2}+yz^{2}+zx^{2})}\geq ...$ (Đọan này áp dụng bổ đề trên)




#629856 $P=\frac{x^4}{y+z}+\frac{y^4}...

Đã gửi bởi tquangmh on 27-04-2016 - 20:07 trong Bất đẳng thức và cực trị

Có cho x, y, z dương hay không âm không vậy bạn ?




#629555 $\sqrt{x+1}+\sqrt{y+1}+\sqrt{z+1...

Đã gửi bởi tquangmh on 25-04-2016 - 20:11 trong Bất đẳng thức và cực trị

 

BĐT được đưa về dạng:

 

$\sqrt{\frac{2a}{a+b}}+\sqrt{\frac{2b}{b+c}}+\sqrt{\frac{2c}{c+a}} \leq 3$         ($a,b,c>0$)

 

 

$\sqrt{\frac{2a}{a+b}}+\sqrt{\frac{2b}{b+c}}+\sqrt{\frac{2c}{c+a}}\leq \sqrt{\left [ 2\sum \left (a+b \right ) \right ]\left [ \sum \frac{2a}{(a+b)(c+a)} \right ]}=\sqrt{\frac{8(a+b+c)(ab+bc+ca)}{(a+b)(b+c)(c+a)}}\leq 3$                                  (C-S & AM-GM)

 

 

 

 

 

 

 

Cho em hỏi là cách nào anh Huyện lại đưa BDT cần cm về BDT đó ạ ?? 




#629550 Kết quả thi toán violympic trên toàn quốc

Đã gửi bởi tquangmh on 25-04-2016 - 19:59 trong Tài liệu - Đề thi

Có phải mỗi tỉnh lấy 25 người mỗi khối không?

 

Em ko biết anh ạ ! ... Em chỉ biết thi và xem kết quả thôi anh ! Vậy nên anh có thể lên fb mà hỏi.  :D




#629545 Kết quả thi toán violympic trên toàn quốc

Đã gửi bởi tquangmh on 25-04-2016 - 19:51 trong Tài liệu - Đề thi

Tại sao lại có lớp 9, mình tưởng chỉ có lớp 4,8,11 thôi chứ

 

Em chẳng biết nữa, anh lên fb của bạn Thần Đồng Toán Học để xem chi tiết. Để em sửa lại tiêu đề.




#629534 Kết quả thi toán violympic trên toàn quốc

Đã gửi bởi tquangmh on 25-04-2016 - 18:55 trong Tài liệu - Đề thi

Đây là link tổng kết kết quả thi toán violympic. (Thông báo chính thức)

http://violympic.vn/Page_New_Detail.aspx?ID=1618




#629489 CM: $\frac{ab}{c}+\frac{bc}...

Đã gửi bởi tquangmh on 25-04-2016 - 12:13 trong Bất đẳng thức và cực trị

Cho a,b,c>0.CM: $\frac{ab}{c}+\frac{bc}{a}+\frac{ac}{b}\geq a+b+c$

 

_ Áp dụng Cauchy  : $\frac{ab}{c}+\frac{bc}{a}\geq 2 \sqrt{\frac{ab}{c}.\frac{bc}{a}}=2b$

Tương tự, cộng lại theo vế, có đpcm.

Dấu "=" khi a = b = c




#629488 CM: \frac{a^3+b^3}{2ab}+\frac{b^3+c^3...

Đã gửi bởi tquangmh on 25-04-2016 - 12:10 trong Bất đẳng thức và cực trị

_ Ta có BDT : $a^{3}+b^{3}\geq ab(a+b)\Rightarrow \frac{a^{3}+b^{3}}{2ab}\geq \frac{a+b}{2}$

Tương tự cộng lại ta có d0pcm.

_ Dấu "=" khi a = b = c.

* Chứng minh bằng tương đương : $a^{3}+b^{3}\geq ab(a+b)\Leftrightarrow (a+b)(a^{2}-ab+b^{2})-ab(a+b)\geq 0 \Leftrightarrow (a+b)(a-b)^{2}\geq 0$

 (a,b,c dương nên BDT luôn đúng)